How to find limit of sin^2(x)/x^2 as x--> infinity

In summary, Dale was trying to solve a few limits problems but was having difficulty. He was able to solve one by using L'Hopital's rule, but the other two were harder and he was still struggling. He asked for help and was given a link to a number 4. In that number, Dale was able to find the answer to the other two problems by using the Fundamental Theorem of Calculus.
  • #1
scorpa
367
1
Hello Everyone,
I'm doing some practice problems and I am stuck on the following limits.
1) Limit as x approaches infinity of (sin^2(x))/x^2
Now for this one I thought that the answer would just be 1 seeing as the limit of sinx/x is 1, however the answer is apparently 0 and I can't see the reasoning for this.
2) Limit as x approaches 0 of (cosx-1)/xsinx
Now for this one I first tried getting an the identity for sin^2x out of it by squaring it but that didn't work. Then I just tried breaking it up, but I just keep ending up getting 0 but apparently the answer is 1/2.
3) Limit as x approaches pi/3 of (cosx-1/2)/(x-pi/3)
Again I am totally lost. I keep wanting to try and get an identity out of it but I just can't get anything useful.
4) Now this question I know should be easy. I am just having trouble understanding where to start. If F(x) = the integral of f(t)dt (the values on the integral are b=x and a=1) and f(t)= integral (squareroot(1+u^2))/u *du . Find F''(3).
Apparently the answer to this is (2squareroot82)/3
If this isn't clear I can always draw it out and post it, I'm sure that what I tryed to type isn't overly clear.
Sorry about these questions guys, I know I haven't exactly shown a lot of work, but these are the ones that I have no idea of how to do, I've been attempting them for 2 days now and just can't seem to get anywhere :frown:
 
Physics news on Phys.org
  • #2
scorpa said:
1) Limit as x approaches infinity of (sin^2(x))/x^2
Now for this one I thought that the answer would just be 1 seeing as the limit of sinx/x is 1, however the answer is apparently 0 and I can't see the reasoning for this.
The limit of [tex]\frac{\sin (x)}{\Mfunction{x}}[/tex] is only 1 as x approaches 0, not as x approaches infinity. This problem is unrelated to that one.

The key to this one is to realize that [tex]\frac{{\sin (x)}^2}{x^2}[/tex]is always between [tex]\frac{1}{x^2}[/tex] and 0.

I think you can use L'hopital's (sp?) rule for 2 and 3.

-Dale
 
Last edited:
  • #3
Hmmm ok, the way I interpreted that problem was in the form of (sinx)(sinx)/x^2 is this wrong? I always thought that sin^2x =(sinx)(sinx) and sinx^2/x^2 =[tex]\frac{{\sin (x)}^2}{x^2}[/tex]
 
  • #4
Sorry about the confusion:
[tex]{\sin (x)}^2 = \sin (x)\,\sin (x) =
{\sin }^2 (x)[/tex]
Mathematica just uses the notation that I used while most textbooks favor your notation.

Either way, you are looking at the wrong part, in one case (sinx/x) you have the limit as x approaches 0, in the other (this problem) you have the limit as x approaches infinity. The squares have nothing to do with it in this case.

-Dale
 
Last edited:
  • #5
Oh ok I see, my mistake I see what you are getting at now. I am still having trouble with the other questions though, I just don't see where to start :(
 
  • #6
scorpa said:
I am still having trouble with the other questions though, I just don't see where to start :(
2 and 3 are just standard L'hopital's rule problems. Just apply his rule until you don't get 0/0.

I didn't really follow 4. I might be able to help if you could TEX it, but I might still be lost.

-Dale
 
  • #7
I will try and write it out and paste a link to number 4. As for 2 and 3 I don't actually know that rule so I will look it up on the net and see what I can come up with from that. Thanks
 
  • #8
Ok so with the L'Hopitals rule, all you have to do is take the derivative of the numerator and denominator and keep doing it until you don't get 0/0 anymore>? Does this always work like this? Can I use this rule all the time in a situation like this? Based on reading about the rule this is how I solved the questions and I ended up with the right answer:

2) cox-1/xsinx ---> -sinx/(sinx-xcosx)---> -cosx/cosx+cosx-xsinx which when you substitute in x=0 gives you an answer of -1/2!

3) (cosx-1/2)/(x-pi/3) -----> -sinx--->-sin(pi/3)---> -sqrt3/2!

If I can use that method all the time that is awesome! I never knew about it before, in class we were never shown that!

Thanks a lot!
 
  • #9
scorpa said:
If [tex]F(x)= \int_1^x f(t)dt[/tex] and [tex]f(t)= \int_1^{t^2} \frac{1+ u^2}{u}du[/itex],
find F'(3)[/tex]

Do you know the "Fundamental Theorem of Calculus": If [tex]F(x)= \int_1^x f(t)dt[/tex] the F'(x)=f(x). In particular, [tex]F'(3)=f(3)= \int_1^{9} \frac{1+ u^2}{u}du[/itex].

To do that integral, multiply both numerator and denominator by u to get
[tex]F'(3)= f(3)= \int_1^{9} \frac{u(1+ u^2)}{u^2}du[/itex], and let v= 1+ u^2.
 
  • #10
scorpa said:
Ok so with the L'Hopitals rule, all you have to do is take the derivative of the numerator and denominator and keep doing it until you don't get 0/0 anymore>? Does this always work like this? Can I use this rule all the time in a situation like this? Based on reading about the rule this is how I solved the questions and I ended up with the right answer:
2) cox-1/xsinx ---> -sinx/(sinx-xcosx)---> -cosx/cosx+cosx-xsinx which when you substitute in x=0 gives you an answer of -1/2!
3) (cosx-1/2)/(x-pi/3) -----> -sinx--->-sin(pi/3)---> -sqrt3/2!
If I can use that method all the time that is awesome! I never knew about it before, in class we were never shown that!
Thanks a lot!

See you were never shown that in class, I would recommend to only use it as a test. Your teacher probably expects you to find it through another route.
 
  • #11
HallsofIvy said:
Do you know the "Fundamental Theorem of Calculus": If [tex]F(x)= \int_1^x f(t)dt[/tex] the F'(x)=f(x). In particular, [tex]F'(3)=f(3)= \int_1^{9} \frac{1+ u^2}{u}du[/itex].
To do that integral, multiply both numerator and denominator by u to get
[tex]F'(3)= f(3)= \int_1^{9} \frac{u(1+ u^2)}{u^2}du[/itex], and let v= 1+ u^2.

Where does the square root go in that integral? And how do you get the second derivative from that? Stupid questions I know, but I'm just now seeing this one .


And how would you go about solving the limit problems without L'hopitals rule? Now that I learned it I rather like it...it's very handy!
 
  • #12
scorpa said:
Ok so with the L'Hopitals rule, all you have to do is take the derivative of the numerator and denominator and keep doing it until you don't get 0/0 anymore>? Does this always work like this? Can I use this rule all the time in a situation like this?
There are (of course) some limitations. If you are trying to take the limit of some f(x)/g(x) then the most important limitation is that the limit of f'(x)/g'(x) must exist.

For example, let [tex]f(x) = \sin (x)[/tex] and [tex]g(x) = x[/tex] then [tex]\Mfunction{Limit}(\frac{f(x)}{g(x)},
{x\rightarrow {\infty}}) = 0[/tex]. However, [tex]f'(x) = \cos (x)[/tex] and [tex]g'(x) = 1[/tex] so [tex]\Mfunction{Limit}(\frac{f'(x)}{g'(x)},
{x\rightarrow {\infty}})[/tex] does not exist. So for this one L'Hopital's rule doesn't work. Instead you have to use the same method as in problem 1 to get the right answer.

-Dale
 
Last edited:
  • #13
DaleSpam said:
There are (of course) some limitations. If you are trying to take the limit of some f(x)/g(x) then the most important limitation is that the limit of f'(x)/g'(x) must exist.
For example, let [tex]f(x) = \sin (x)[/tex] and [tex]g(x) = x[/tex] then [tex]\Mfunction{Limit}(\frac{f(x)}{g(x)},
{x\rightarrow {\infty}}) = 0[/tex]. However, [tex]f'(x) = \cos (x)[/tex] and [tex]g'(x) = 1[/tex] so [tex]\Mfunction{Limit}(\frac{f'(x)}{g'(x)},
{x\rightarrow {\infty}})[/tex] does not exist. So for this one L'Hopital's rule doesn't work. Instead you have to use the same method as in problem 1 to get the right answer.
-Dale
?
What is this? :confused:
What do you mean?
---------------------
L'Hopital rule is used when both numerator and denominator either tends to 0 or infinity, i.e:[tex]\frac{0}{0} \quad \mbox{or} \quad \frac{\infty}{\infty}[/tex]. It states that, if:
[tex]\lim_{x \rightarrow \alpha} \frac{f(x)}{g(x)}[/tex] exists, and either:
[tex]\lim_{x \rightarrow \alpha} f(x) = 0 \quad \mbox{and} \quad \lim_{x \rightarrow \alpha} g(x) = 0[/tex] or:
[tex]\lim_{x \rightarrow \alpha} f(x) = \infty \quad \mbox{and} \quad \lim_{x \rightarrow \alpha} g(x) = \infty[/tex], then:
[tex]\lim_{x \rightarrow \alpha} \frac{f(x)}{g(x)} = \lim_{x \rightarrow \alpha} \frac{f'(x)}{g'(x)}[/tex].
-------------------
#1, you cannot use L'Hopital rule since you don't know what sin2(x) tends to when x tends to infinity. It can be done using Squeeze theorem, by noticing the fact that:
[tex]0 \leq \frac{\sin ^ 2 x}{x ^ 2} \leq \frac{1}{x ^ 2}[/tex]
-------------------
#2, you have two different ways to do it without using L'Hopital rule:
The first way is to use power-reduction formulas:
[tex]\sin ^ 2 x = \frac{1 - \cos(2x)}{2}[/tex].
So here you have:
[tex]\lim_{x \rightarrow 0} \frac{\cos x - 1}{x \sin x} = \lim_{x \rightarrow 0} - \frac{2 \sin ^ 2 \left( \frac{x}{2} \right)}{x \sin x} = \lim_{x \rightarrow 0} - \frac{2 \sin ^ 2 \left( \frac{x}{2} \right)}{4 \frac{x}{2} \sin \left( \frac{x}{2} \right) \cos \left( \frac{x}{2} \right)}[/tex].
Can you go from here?
The second way is to multiply both numerator and denominator by (1 + cos x), so that you'll have sin2x in the numerator.
It goes like this:
[tex]\lim_{x \rightarrow 0} \frac{\cos x - 1}{x \sin x} = \lim_{x \rightarrow 0} \frac{(\cos x - 1)(1 + \cos x)}{x \sin x (1 + \cos x)} = \lim_{x \rightarrow 0} \frac{- \sin ^ 2 x}{x \sin x (1 + \cos x)}[/tex].
Can you go from here?
-------------------
#3, you can change 1 / 2 into: [tex]\frac{1}{2} = \cos \left( \frac{\pi}{3} \right)[/tex], then use the identity:
[tex]\cos \alpha - \cos \beta = -2 \sin \left( \frac{\alpha + \beta}{2} \right) \sin \left( \frac{\alpha - \beta}{2} \right)[/tex]. Can you go from here?
L'Hopital rule is overkill... and as a matter of fact, you haven't learned it, so you shouldn't use it.
--------------------
If [tex]F(x) := \int_{\alpha} ^ {x} f(t) dt[/tex] then [tex]F'(x) = \frac{d \left( \int_{\alpha} ^ {x} f(t) dt \right)}{dx} = f(x)[/tex]. There should be a proof in your book, or you can view it here (see part 3).
So [tex]F''(x) = f'(x) = \frac{d \left( \int_{1} ^ {x ^ 2} f(u) du \right)}{dx} \ = \ ?[/tex], where [tex]f(u) := \frac{\sqrt{1 + u ^ 2}}{u}[/tex]. Note that the upper limit of the integral is x2, not x.
Can you go from here?
 
Last edited:
  • #14
OK, thanks so much! I think I understand those limits now even without that handy little rule. And I can now get the answer for that integral! Thanks very much guys!
 

1. What is the limit of sin^2(x)/x^2 as x approaches infinity?

The limit of sin^2(x)/x^2 as x approaches infinity is equal to 0. This can be solved by using the squeeze theorem, where the numerator is bounded by 1, and the denominator approaches infinity.

2. How do I solve for the limit of sin^2(x)/x^2 as x approaches infinity?

To solve for this limit, you can use the L'Hopital's rule, where you take the derivatives of both the numerator and denominator until you get a finite value. In this case, the derivative of sin^2(x) is 2sin(x)cos(x) and the derivative of x^2 is 2x. Plugging these values into the limit equation, you will get the answer of 0.

3. Can I use a graph to find the limit of sin^2(x)/x^2 as x approaches infinity?

Yes, you can use a graph to find the limit of sin^2(x)/x^2. When graphing this function, you will notice that as x approaches infinity, the graph approaches the x-axis, indicating a limit of 0.

4. Is there a specific method to solve for the limit of sin^2(x)/x^2 as x approaches infinity?

Yes, there are multiple methods that can be used to solve for this limit. Some common methods include using the squeeze theorem, L'Hopital's rule, and graphing the function. Choose the method that you are most comfortable with and that suits the given problem.

5. Can the limit of sin^2(x)/x^2 as x approaches infinity have a different value?

No, the limit of sin^2(x)/x^2 as x approaches infinity will always be 0. This is because the numerator (sin^2(x)) approaches 0 and the denominator (x^2) approaches infinity as x gets larger. Therefore, the limit will always be 0 regardless of the value of x.

Similar threads

  • Calculus and Beyond Homework Help
Replies
3
Views
339
  • Calculus and Beyond Homework Help
Replies
12
Views
778
  • Calculus and Beyond Homework Help
Replies
17
Views
604
  • Calculus and Beyond Homework Help
Replies
8
Views
658
  • Calculus and Beyond Homework Help
Replies
26
Views
2K
  • Calculus and Beyond Homework Help
Replies
20
Views
1K
  • Calculus and Beyond Homework Help
Replies
5
Views
1K
  • Calculus and Beyond Homework Help
Replies
6
Views
545
  • Calculus and Beyond Homework Help
Replies
24
Views
2K
  • Calculus and Beyond Homework Help
Replies
5
Views
169
Back
Top